2
0
Fork 0
mirror of https://github.com/MartinThoma/LaTeX-examples.git synced 2025-04-18 19:18:21 +02:00
LaTeX-examples/documents/GeoTopo/Loesungen.tex
Martin Thoma 7740f0147f Remove trailing spaces
The commands

find . -type f -name '*.md' -exec sed --in-place 's/[[:space:]]\+$//' {} \+

and

find . -type f -name '*.tex' -exec sed --in-place 's/[[:space:]]\+$//' {} \+

were used to do so.
2015-10-14 14:25:34 +02:00

355 lines
No EOL
18 KiB
TeX

%!TEX root = GeoTopo.tex
\chapter*{Lösungen der Übungsaufgaben\markboth{Lösungen der Übungsaufgaben}{Lösungen der Übungsaufgaben}}
\addcontentsline{toc}{chapter}{Lösungen der Übungsaufgaben}
\begin{solution}[\ref{ub1:aufg1}]
\textbf{Teilaufgabe a)} Es gilt:
\begin{enumerate}[label=(\roman*)]
\item $\emptyset, X \in \fT_X$.
\item $\fT_X$ ist offensichtlich unter Durchschnitten abgeschlossen,
d.~h. es gilt für alle $U_1, U_2 \in \fT_X: U_1 \cap U_2 \in \fT_X$.
\item Auch unter beliebigen Vereinigungen ist $\fT_X$ abgeschlossen,
d.~h. es gilt für eine beliebige Indexmenge $I$ und alle
$U_i \in \fT_X$ für alle $i \in I: \bigcup_{i \in I} U_i \in \fT_X$
\end{enumerate}
Also ist $(X, \fT_X)$ ein topologischer Raum.
\textbf{Teilaufgabe b)} Wähle $x=1, y=0$. Dann gilt $x \neq y$
und die einzige Umgebung von $x$ ist $X$. Da $y=0 \in X$ können
also $x$ und $y$ nicht durch offene Mengen getrennt werden.
$(X, \fT_X)$ ist also nicht hausdorffsch.
\textbf{Teilaufgabe c)} Nach Bemerkung \ref{Trennungseigenschaft}
sind metrische Räume hausdorffsch. Da $(X, \fT_X)$ nach (b) nicht
hausdorffsch ist, liefert die Kontraposition der Trennungseigenschaft,
dass $(X, \fT_X)$ kein metrischer Raum sein kann.
\end{solution}
\begin{solution}[\ref{ub1:aufg4}]
\textbf{Teilaufgabe a)}
\textbf{Beh.:} $\forall a \in \mdz: \Set{a}$ ist abgeschlossen.
Sei $a \in \mdz$ beliebig. Dann gilt:
Wenn jemand diese Aufgabe gemacht hat, bitte die Lösung an info@martin-thoma.de
schicken.%TODO
\textbf{Teilaufgabe b)}
\textbf{Beh.:} $\Set{-1, 1}$ ist nicht offen
\textbf{Bew.:} durch Widerspruch
Annahme: $\Set{-1, 1}$ ist offen.
Dann gibt es $T \subseteq \fB$, sodass $\bigcup_{M \in T} M = \Set{-1, 1}$.
Aber alle $U \in \fB$ haben unendlich viele Elemente. Auch endlich
viele Schnitte von Elementen in $\fB$ haben unendlich viele
Elemente $\Rightarrow$ keine endliche nicht-leere Menge kann
in dieser Topologie offen sein $\Rightarrow \Set{-1,1}$ ist
nicht offen. $\qed$
\textbf{Teilaufgabe c)}
\textbf{Beh.:} Es gibt unendlich viele Primzahlen.
\textbf{Bew.:} durch Widerspruch
Annahme: Es gibt nur endlich viele Primzahlen $p \in \mdp$
Dann ist
\[\mdz \setminus \Set{-1, +1} \overset{\text{FS d. Arithmetik}}= \bigcup_{p \in \mdp} U_{0,p}\]
endlich. Das ist ein Widerspruch zu $|\mdz|$ ist unendlich und
$|\Set{-1,1}|$ ist endlich. $\qed$
\end{solution}
\begin{solution}[\ref{ub2:aufg4}]
\begin{enumerate}[label=(\alph*)]
\item \textbf{Beh.:} Die offenen Mengen von $P$ sind
Vereinigungen von Mengen der Form
\[\prod_{j \in J} U_j \times \prod_{i \in \mdn, i \neq j} P_i\]
wobei $J \subseteq \mdn$ endlich und $U_j \subseteq P_j$
offen ist.
\begin{beweis}
Nach Definition der Produkttopologie bilden Mengen
der Form
\[\prod_{i \in J} U_j \times \prod_{i \in \mdn \setminus J} P_i\]
wobei $J \subseteq \mdn$ endlich und $U_j \subseteq P_j$ offen
$\forall{j \in J}$
eine Basis der Topologie.
Damit sind die offenen
Mengen von $P$ Vereinigungen von Mengen der obigen
Form. $\qed$
\end{beweis}
\item \textbf{Beh.:} Die Zusammenhangskomponenten von $P$
sind alle einpunktig.\xindex{Total Unzusammenhängend}
\begin{beweis}
Es seinen $x,y \in P$ und $x$ sowie $y$ liegen in der
gleichen Zusammenhangskomponente $Z \subseteq P$.
Da $Z$ zusammenhängend ist und $\forall{i \in I}: p_i : P \rightarrow P_i$
ist stetig, ist $p_i(Z) \subseteq P_i$ zusammenhängend
für alle $i \in \mdn$. Die zusammenhängenden Mengen
von $P_i$ sind genau $\Set{0}$ und $\Set{1}$, d.~h.
für alle $i \in \mdn$ gilt entweder $p_i(Z) \subseteq \Set{0}$
oder $p_i(Z) \subseteq \Set{1}$. Es sei $z_i \in \Set{0,1}$
so, dass $p_i(Z) \subseteq \Set{z_i}$ für alle $i \in \mdn$.
Dann gilt also:
\[\underbrace{p_i(x)}_{= x_i} = z_i = \underbrace{p_i(y)}_{= y_i} \forall i \in \mdn\]
Somit folgt: $x = y \qed$
\end{beweis}
\end{enumerate}
\end{solution}
\begin{solution}[\ref{ub3:aufg1}]
\begin{enumerate}[label=(\alph*)]
\item \textbf{Beh.:} $\GL_n(\mdr)$ ist nicht kompakt.\\
\textbf{Bew.:} $\det: \GL_n(\mdr) \rightarrow \mdr \setminus \Set{0}$
ist stetig. Außerdem ist
$\det(\GL_n(\mdr)) = \mdr \setminus \Set{0}$ nicht
kompakt. $\overset{\ref{kor:5.6}}{\Rightarrow}$
$\GL_n(\mdr)$ ist nicht kompakt. $\qed$
\item \textbf{Beh.:} $\SL_1(\mdr)$ ist nicht kompakt, für $n > 1$ ist $\SL_n(\mdr)$ kompakt.\\
\textbf{Bew.:} Für $\SL_1(\mdr)$ gilt:
$\SL_1(\mdr) = \Set{A \in \mdr^{1 \times 1} | \det A = 1} = \begin{pmatrix}1\end{pmatrix} \cong \Set{1}$.
$\overset{\ref{kor:5.6}}{\Rightarrow} \SL_1(\mdr)$ ist
kompakt.\\
$\SL_n(\mdr) \subseteq \GL_n(\mdr)$ lässt sich mit einer
Teilmenge des $\mdr^{n^2}$ identifizieren. Nach \cref{satz:heine-borel}
sind diese genau dann kompakt, wenn sie beschränkt und
abgeschlossen sind. Definiere nun für für $n \in \mdn_{\geq 2}, m \in \mdn$:
\[A_m = \text{diag}_n(m, \frac{1}{m}, \dots, 1)\]
Dann gilt: $\det A_m = 1$, d.~h. $A_m \in \SL_n(\mdr)$,
und $A_m$ ist unbeschränkt, da $\|A_m\|_\infty =m \xrightarrow[m \rightarrow \infty]{} \infty$.$\qed$
\item \textbf{Beh.:} $\praum(\mdr)$ ist kompakt.\\
\textbf{Bew.:} $\praum(\mdr) \cong S^n/_{x \sim -x}$.
Per Definition der Quotiententopologie ist die Klassenabbildung stetig.
Da $S^n$ als abgeschlossene und beschränkte Teilmenge
des $\mdr^{n+1}$ kompakt ist $\overset{\ref{kor:5.6}}{\Rightarrow}$
$\praum(\mdr)$ ist kompakt. $\qed$
\end{enumerate}
\end{solution}
\begin{solution}[\ref{ub3:meinsExtra}]
Die Definition von Homöomorphismus kann auf \cpageref{def:homoeomorphismus}
nachgelesen werden.
\begin{definition}\xindex{Homomorphismus}%
Seien $(G, *)$ und $(H, \circ)$ Gruppen und
$\varphi:G \rightarrow H$ eine Abbildung.
$\varphi$ heißt \textbf{Homomorphismus}, wenn
\[\forall g_1, g_2 \in G: \varphi(g_1 * g_2) = \varphi(g_1) \circ \varphi(g_2)\]
gilt.
\end{definition}
Es folgt direkt:
\begin{bspenum}
\item Sei $X = \mdr$ mit der Standarttopologie und $\varphi_1: \id_\mdr$ und $\mdr = (\mdr,+)$. Dann ist $\varphi_1$ ein Gruppenhomomorphismus und ein Homöomorphismus.
\item Sei $G = (\mdz, +)$ und $H = (\mdz / 3 \mdz, +)$. Dann ist $\varphi_2 : G \rightarrow H, x \mapsto x \mod 3$ ein Gruppenhomomorphismus.
Jedoch ist $\varphi_2$ nicht injektiv, also sicher kein Homöomorphismus.
\item Sei $X$ ein topologischer Raum. Dann ist $\id_X$ ein Homöomorphismus. Da keine Verknüpfung auf $X$ definiert wurde, ist $X$ keine Gruppe und daher auch kein Gruppenhomomorphismus.
\end{bspenum}
Also: Obwohl die Begriffe ähnlich klingen, werden sie in ganz unterschiedlichen
Kontexten verwendet.
\end{solution}
\begin{solution}[\ref{ub3:meinsExtra2}]
Die Definition einer Isotopie kann auf \cpageref{def:Isotopie} nachgelesen
werden, die einer Isometrie auf \cpageref{def:Isometrie}.
\begin{definition}\xindex{Isomorphismus}%
Seien $(G, *)$ und $(H, \circ)$ Gruppen und
$\varphi:G \rightarrow H$ eine Abbildung.
$\varphi$ heißt \textbf{Isomorphismus}, wenn $\varphi$ ein bijektiver
Homomorphismus ist.
\end{definition}
Eine Isotopie ist also für Knoten definiert, Isometrien machen nur in
metrischen Räumen Sinn und ein Isomorphismus benötigt eine Gruppenstruktur.
\end{solution}
\begin{solution}[\ref{ub4:aufg1}]
\begin{enumerate}[label=(\alph*)]
\item \textbf{Vor.:} Sei $M$ eine topologische Mannigfaltigkeit.\\
\textbf{Beh.:} $M$ ist wegzusammehängend $\gdw M$ ist zusammenhängend
\begin{beweis}
\enquote{$\Rightarrow$}: Da $M$ insbesondere ein
topologischer Raum ist folgt diese Richtung direkt
aus \cref{kor:wegzusammehang-impliziert-zusammenhang}.
\enquote{$\Leftarrow$}: Seien $x,y \in M$ und
\[Z := \Set{z \in M | \exists \text{Weg von } x \text{ nach } z}\]
Es gilt:
\begin{enumerate}[label=(\roman*)]
\item $Z \neq \emptyset$, da $M$ lokal wegzusammenhängend ist
\item $Z$ ist offen, da $M$ lokal wegzusammenhängend ist
\item $Z^C := \Set{\tilde{z} \in M | \nexists \text{Weg von } x \text{ nach } \tilde{z}}$ ist offen
Da $M$ eine Mannigfaltigkeit ist, existiert zu jedem
$\tilde{z} \in Z^C$ eine offene und wegzusammenhängende Umgebung
$U_{\tilde{z}} \subseteq M$.
Es gilt sogar $U_{\tilde{z}} \subseteq Z^C$, denn
gäbe es ein $U_{\tilde{z}} \ni \overline{z} \in Z$,
so gäbe es Wege $\gamma_2:[0,1] \rightarrow M, \gamma_2(0) = \overline{z}, \gamma_2(1) = x$
und $\gamma_1:[0,1] \rightarrow M, \gamma_1(0) = \tilde{z}, \gamma_1(1) = \overline{z}$.
Dann wäre aber
\begin{align*}
\gamma:[0,1] &\rightarrow M,\\
\gamma(x) &= \begin{cases}
\gamma_1(2x) &\text{falls } 0 \leq x \leq \frac{1}{2}\\
\gamma_2(2x-1) &\text{falls } \frac{1}{2} < x \leq 1
\end{cases}
\end{align*}
ein stetiger Weg von $\tilde{z}$ nach $x$
$\Rightarrow$ Widerspruch.
Da $M$ zusammenhängend ist und $M = \underbrace{Z}_{\mathclap{\text{offen}}} \cup \underbrace{Z^C}_{\mathclap{\text{offen}}}$,
sowie $Z \neq \emptyset$ folgt $Z^C = \emptyset$.
Also ist $M=Z$ wegzusammenhängend.$\qed$
\end{enumerate}
\end{beweis}
\item \textbf{Beh.:} $X$ ist wegzusammenhängend.\\
\begin{beweis}
$X:= (\mdr \setminus \Set{0}) \cup \Set{0_1, 0_2}$
und $(\mdr \setminus \Set{0}) \cup \Set{0_2}$ sind
homöomorph zu $\mdr$. Also sind die einzigen kritischen
Punkte, die man nicht verbinden können könnte
$0_1$ und $0_2$.
Da $(\mdr \setminus \Set{0}) \cup \Set{0_1}$ homöomorph
zu $\mdr$ ist, exisitert ein Weg $\gamma_1$ von $0_1$
zu einem beliebigen Punkt $a \in \mdr \setminus \Set{0}$.
Da $(\mdr \setminus \Set{0}) \cup \Set{0_2}$ ebenfalls
homöomorph zu $\mdr$ ist, existiert außerdem ein Weg
$\gamma_2$ von $a$ nach $0_2$. Damit existiert ein
(nicht einfacher)
Weg $\gamma$ von $0_1$ nach $0_2$. $\qed$
\end{beweis}
\end{enumerate}
\end{solution}
%Das scheint mir etwas zu lang zu sein...
%\begin{solution}[\ref{ub7:aufg1}]
% \textbf{Beh.:} $H_k = \begin{cases}\mdr &\text{für } k\in \Set{0,1}\\
% 0 &\text{für } k \geq 2$
% \newcommand{\triangleSimplizialkomplex}{\mathord{\includegraphics[height=5ex]{figures/triangleSimplizialkomplex.pdf}}}
% \textbf{Bew.:} $S^1$ ist homöomorph zum Simplizialkomplex
% $X = \triangleSimplizialkomplex$, d.~h. dem Rand
% von $\Delta^2$. Es gilt:
% \[X = \Set{\underbrace{v_0, v_1, v_2}_{A_0(X)}, \underbrace{\Delta (v_1, v_2)}_{=: a_0}, \underbrace{\underbrace{\Delta (v_0, v_2)}_{=: a_1}, \underbrace{\Delta(v_0, v_1)}_{=: a_2}}_{A_1(X)}}\]
% Damit folgt:
% \begin{enumerate}
% \item Für $k \geq 2$ ist $C_k(X) \cong 0$, da es in diesen
% Dimensionen keine Simplizes gibt, d.~h. $A_k(X) = \emptyset$ gilt.\\
% Also: $H_k(X) \cong 0 \; \forall k \geq 2$
% \item $C_0(X) = \Set{\sum_{i=0}^2 c_i v_i | c_i \in \mdr}$, da
% $A_0(x)$ Basis von $C_0(X)$ ist;\\
% $C_1(X) = \Set{\sum_{i=0}^2 c_i a_i | c_i \in \mdr}$, da
% $A_1(X)$ Basis von $C_1(X)$ ist.
% \item Für die Randabbildungen $d_i: C_i(X) \rightarrow C_{i-1}(X)$ gilt:
% $d_0 \equiv 0$, $d_1: C_1(X) \rightarrow C_0(X)$ ist definiert durch
% $d_1(a_k) = \sum_{i=0}^1 (-1)^i \partial_i(a_k) = \partial_0 (a_k) - \partial_1(a_k) \; \forall k \in \Set{0,1,2}$
% \end{enumerate}
%\end{solution}
%Auch diese Aufgabe ist zu lang
%\begin{solution}[\ref{ub7:aufg3}]
%
%\end{solution}
\begin{solution}[\ref{ub11:aufg3}]
\textbf{Vor.:} Sei $(X, d)$ eine absolute Ebene, $A, B, C \in X$
und $\triangle ABC$ ein Dreieck.
\begin{enumerate}[label=(\alph*)]
\item \textbf{Beh.:} $\overline{AB} \cong \overline{AC} \Rightarrow \angle ABC \cong \angle ACB$\\
\textbf{Bew.:} Sei $\overline{AB} \cong \overline{AC}$.\\
$\Rightarrow \exists$ Isometrie $\varphi$ mit $\varphi(B) = C$ und
$\varphi(C) = B$ und $\varphi(A) = A$.\\
$\Rightarrow \varphi(\angle ABC) = \angle ACB$\\
$\Rightarrow \angle ABC \cong \angle ACB \qed$
\item \textbf{Beh.:} Der längeren Seite von $\triangle ABC$ liegt der größere Winkel gegenüber und
umgekehrt.\\
\textbf{Bew.:} Sei $d(A,C) > d(A,B)$. Nach \ref{axiom:3.1}
gibt es $C' \in AC^+$ mit $d(A, C') = d(A,B)$\\
$\Rightarrow C'$ liegt zwischen $A$ und $C$.\\
Es gilt $\measuredangle ABC' < \measuredangle ABC$ und
aus \cref{ub11:aufg3.a} folgt: $\measuredangle ABC' = \measuredangle AC' B$.\\
$\angle BC' A$ ist ein nicht anliegender Außenwinkel zu
$\angle BCA \xRightarrow{\crefabbr{bem:14.9}} \measuredangle BC' A > \measuredangle BCA$\\
$\Rightarrow \measuredangle BCA < \measuredangle BC' A = \measuredangle ABC' < \measuredangle ABC $
Sei umgekehrt $\measuredangle ABC > \measuredangle BCA$,
kann wegen 1. Teil von \cref{ub11:aufg3.b} nicht
$d(A,B) > d(A,C)$ gelten.\\
Wegen \cref{ub11:aufg3.a} kann nicht $d(A,B) = d(A,C)$
gelten.\\
$\Rightarrow d(A,B) < d(A, C) \qed$
\item \textbf{Vor.:} Sei $g$ eine Gerade, $P \in X$ und $P \notin g$\\
\textbf{Beh.:} $\exists!$ Lot\\
\textbf{Bew.:} ÜB10 A4(a): Es gibt Geradenspiegelung $\varphi$
an $g$. $\varphi$ vertauscht die beiden Halbebenen bzgl.
$g$.\\
$\Rightarrow \varphi(P)P$ schneidet $g$ in $F$.
%Nach ÜB 10 A4(a):
Es gibt eine Geradenspiegelung $\varphi$ an $g$.
$\varphi$ vertauscht die beiden Halbebenen bzgl. $g$
$\Rightarrow \varphi(P)P$ schneidet $g$ in $F$.
Sei $A \in g \setminus \Set{F}$. Dann gilt $\varphi(\angle AFP) = \angle AF \varphi(P) = \pi$
$\Rightarrow \angle AFP$ ist rechter Winkel.
Gäbe es nun $G \in g \setminus \Set{F}$, so dass $PG$ weiteres Lot von $P$ auf $g$ ist,
wäre $\triangle PFG$ ein Dreieck mit zwei rechten Innenwinkeln (vgl. \cref{fig:two-perpendiculars}).
\begin{figure}[htp]
\centering
\input{figures/two-perpendiculars.tex}
\caption{Zwei Lote zu einer Geraden $g$ durch einen Punkt $P$}
\label{fig:two-perpendiculars}
\end{figure}
Nach \cref{folgerung:14.10} ist die Summe von zwei Innenwinkeln immer $< \pi$\\
$\Rightarrow G$ gibt es nicht. $\qed$
\end{enumerate}
\end{solution}
\begin{solution}[\ref{ub-tut-24:a1}]
Sei $f \parallel h$ und \obda $f \parallel g$.
$f \nparallel h \Rightarrow f \cap h \neq \emptyset$, sei also $x \in f \cap h$.
Mit Axiom \ref{axiom:5} folgt: Es gibt höchstens eine Parallele
zu $g$ durch $x$, da $x \notin g$. Diese ist $f$, da $x \in f$
und $f \parallel g$. Da aber $x \in h$, kann $h$ nicht parallel
zu $g$ sein, denn ansonsten gäbe es zwei Parallelen zu $g$ durch
$x$ ($f \neq h$).
$\Rightarrow g \nparallel h$ $\qed$
\end{solution}
\begin{solution}[\ref{ub-tut-24:a3}]\xindex{Kongruenzsatz!SSS}%
Sei $(X,d,G)$ eine Geometrie, die \ref{axiom:1}-\ref{axiom:4} erfüllt.
Seien außerdem $\triangle ABC$ und $\triangle A'B' C'$ Dreiecke, für die gilt:
\begin{align*}
d(A, B) &= d(A', B')\\
d(A, C) &= d(A', C')\\
d(B, C) &= d(B', C')
\end{align*}
Sei $\varphi$ die Isometrie mit $\varphi(A) = A'$, $\varphi(B) = B'$ und
$\varphi(C')$ liegt in der selben Halbebene bzgl. $AB$ wie $C$. Diese
Isometrie existiert wegen \ref{axiom:4}.
Es gilt $d(A,C) = d(A', C') = d(\varphi(A'), \varphi(C')) = d(A, \varphi(C'))$
und $d(B,C) = d(B', C') = d(\varphi(B'), \varphi(C')) = d(B, \varphi(C'))$.\\
$\xRightarrow{\crefabbr{kor:14.6}} C = \varphi(C)$.
Es gilt also $\varphi(\triangle A'B'C') = \triangle ABC$. $\qed$
\end{solution}